Brain and Behaviour Learning Objectives

¡Supera tus tareas y exámenes ahora con Quizwiz!

List the main regions of the brain involved in pain.

1) Anterior cingulate cortex - caudal deals with quality of pain, rostral deals with actions to do as a result of the pain 2) Amygdala labels the memory with an emotional experience of pain 3) Primary somatosensory cortex - discrimination and localisation of pain 4) Prefrontal cortex - evaluation of pain, cognitive aspect of pain 5) Insula - interoception (awareness of physiological state of body), pain map and homeostatic adjustment.

Illustrate the relevance of learning principles in medicine.

1) Counter-conditioning - aversion therapy. Eg using antabuse for alcoholics (makes you feel nausea and vomit), eventually the alcohol causes nausea and vomiting without the antabuse. 2) Systematic desensitisation - Replacing one response with another - changing anxiety into relaxation - patient put through a series of situations increasing their anxiety and at each stage put into practice relaxation techniques they have learnt. 3) Extinction procedures - -Implosive therapy - anxiety cannot be maintained at the same level over time - patient is exhausted/satiated. -Flooding - go through the anxiety-inducing situation to learn it is not harmful.

Understand the role of dopamine in the action of the basal ganglia

1) Dopamine binds D1 receptors on medium spiny neurones in the direct pathway with excitatory effect, causing overall excitation of the motor thalamus/supplementary motor cortex --> allows change in motor pathways. 2) Dopamine binds D2 receptors on medium spiny neurones in the indirect pathway with inhibitory effect --> decreases the action of the indirect pathway.

Describe the mechanisms of action, pharmacokinetics, adverse effects and limitations of the drugs used in the treatment of Parkinson's Disease.

1) Dopamine precursor + Dopamine decarboxylase inhibitor. Eg. L-dopa and carbidopa/benserazide. Together they make madopar. SFX: vomiting, nausea, daytime sleepiness, psychosis, control issues. Motor on/off effect after 3-5 years, wearing off quickly, dyskinesia and dystonia. 2) Dopamine receptor agonist (D1-5 receptors) eg. rotigotine, ropinirole, pramipexole. (Pergolide, bromocriptine and cabergoline can cause cardiac fibrosis). Transdermal patch - good for those who cannot swallow. 3) MAO-B inhibitor eg. selegiline, rasagiline. Stops breakdown of dopamine to DOPAC. First-line treatment. 4) COMT inhibitor, tolcapone, entacapone. Stops breakdown of DOPAC into homovanillic acid. Given with L-dopa to increase efficacy. 5) Apomorphine (Strong dopamine receptor agonist). "Apo-go". Binds to dopamine receptors to elicit dopaminergic effects in the nigrostriatal pathways. Infused/injected - good for those who cannot swallow. 6) Amantadine (NMDA receptor antagonist) - weak therapy. Prevents dopamine re-uptake, increases dopamine release. 7) Anti-cholinergics - dopamine loss leads to hyperactivity of cholinergic cells. 8) Deep brain stimulation - lesions can be made/stimulation of subthalamic nucleus, GPi, motor thalamus. Strict criteria for use - no depression, no dementia.

What are some alternative uses of deep brain stimulation other than to treat Parkinson's?

1) Dystonia - DYT-1 dystonia is the most common generalised form. If botox isn't helpful then DBS. 2) Tremors - similar sites of insertion as Parkinson's 3) Chronic pain - targets sensory thalamic nucleus or periaqueductal grey. 4) Depression - nucleus accumbens, subgenial cingulate gyrus. 5) Epilepsy - anterior thalamic nucleus - Alzheimer's, schizophrenia 6) Tourette's.

To discuss the main treatment approaches and their limitations

1) Harm reduction/drug substitutes - methadone for heroin addicts removes them from criminal situations. Nicotine replacement therapies for smokers. 2) Detoxification - need the person to want to come off the drug "cold turkey" and they are no longer driven by the need for it after 4-5 weeks, this only really works in smoking 3) Aversion therapies - antabuse (disulfiram) 4) Motivational therapies - AA/NA groups 5) CBT - teaches patient coping skills and offers new perspectives 6) Treatments for withdrawal relief - drugs to occupy receptors that addiction normally hits - nicotine replacement therapy.

Give the criteria for deep brain stimulation in Parkinson's Disease patients.

1) Idiopathic Parkinson's disease - not pyramidal dysfunction/other mimicking syndromes 2) Patient has not responded well to pharmacological therapy 3) Patient must be in sufficient health/cognitive state. No dementia, no depression. 4) Levodopa challenge test to check that it is not "burnt-out" Parkinson's (test patient without medication "off" and "on" with medication). 5) Must have MRI to check that target is clearly delineated 6) Patient needs to understand that deep brain stimulation does not cure Parkinson's but can possibly improve symptoms.

Describe the process of deep brain stimulation and its results/complications.

1) Microelectrode insertion via frontal burr hole, brain region stimulated and response checked. 2) Pulse generator insertion - permanent electrode and connected to neurostimulator 3) DBS adjustment Results: shown to reduced bradykinesia and dyskinesia. Medications can be reduced by up to 50%. Patient has more "on" time - control of motor actions. Complications: infections, lead migration, fracture, delayed intracranial haemorrhage Reversible: depression, speech defects/disturbances, unsteadiness.

Describe the histology of multiple sclerosis.

1) Normal white matter with a lesion - looks like a black hole. 2) Pre-active lesion - activation of microglia and macrophages 3) Chronic active lesion - immune reactions take place (Antibody mediated, T-cell, macrophages). Loss of myelin leads to axonal loss, although attempts at remyelination may be made. 4) Chronic inactive lesion - area fills with brown astrocytic scars. 5) Remyelination - shadowed plaques, some return of red axons and green myelin but very little.

Be able to describe the pathogenesis of Alzheimer's Disease.

1) The Amyloid Cascade Hypothesis: Beta-amyloid plaque formation. Amyloid precursor protein APP (thought to regulate synapse function and neural plasticity in neurones, glial cells etc.) is normally recycled in the brain by a-secretase and y-secretase. When b-secretase cleaves APP it forms non-soluble breakdown products of b-amlyoid peptides which are neurotoxic. The b-amyloid peptides aggregate into soluble oligomers (Ab40/42) which cause neuronal and vascular degeneration through oxidative stress - altered neuronal metabolism and ionic homeostasis --> progressive neuronal injury. The plaques deposit between neurones, preventing normal synaptic transmission causing cognitive and memory issues. There is an inflammatory response of microglial activation, astrocytosis and acute phase protein release. 2) Neurofibrillary tangle production - tau is normally bound to microtubules to stabilise them/keep them straight. Microtubules are cytoskeletal structures along which neuronal fuel substances are shunted eg. ATP. Through mechanisms not clearly understood, the presence of extracellular b-amyloid oligomers is through to activate intracellular kinase --> phosphorylates tau --> tau pulls away from microtubule structure --> microtubules collapse preventing adequate delivery of neuronal fuels. Hyperphosphorylated tau proteins aggregate with paired helical filaments to create neurofibrillary tangles. Recent slues have shown that there s a "contagion" like diffusion of beta-amyloid oligomers/hyperphosphorylated tau oligomers into support astrocytes and oligodendrocytes causing cellular death. The brain slowly atrophies over a course of about 20 years before symptoms appear.

Symptoms of the mind: thought interferences

1) Thought broadcasting - belief that other people can read your thoughts 2) Thought insertion - belief that other people can put thoughts into your head 3) Thought withdrawal - belief the thoughts have been removed from your mind

To examine individual differences in vulnerability to addiction

1) sensitivity to positive effects 2) sensitivity to negative effects 3) personality 4) mental health 5) habitual stress 6) withdrawal symptoms 7) drive caused by drug 8) social/environmental factors Some drugs are more addictive than others - speed of delivery, concentration, presence of other facilitating chemicals.

What is the role of the basal ganglia?

1. The gate between motor programmes 2. inhibit antagonistic movement 3. regulate intensity or slow or stereotyped movement 4. cognition and attention

How is epilepsy diagnosed?

2 or more seizures = epilepsy can be diagnosed. Will have to be based on eyewitness accounts. Brain scans: MRIs, fMRIs, PET scans can be used to look at changes in brain structure and anatomy. EEG, electroencephalography can make recordings of seizures (these have high amplitude on EEG). It is hoped in the future ambulatory EEGs can be used to predict seizures.

Describe the genetics of multiple sclerosis.

A combination of environmental and genetic causes contribute. HLA-DRB1-1501 on chromosome 6 p2 1 is indicated.

Describe the neurotransmitter states during sleep.

ACh released from pedunculopontine nucleus/nucleus basalis in dream state. Noradrenaline not released from locus coeruleus - if it is then you remember your dreams (the connection from the LC to the cerebral cortex is said to be a central sympathetic system). Serotonin neurones are completely inactive - serotonin is thought to be the sensory gate between the external world and internal world. The cortex generates internal images/perceptions from memories because there is no sensory information coming in during sleep. Dopaminergic action is stopped in the brain to stop you from moving in your sleep. Histamines (from tuberomamillary nucleus) are low in sleep - they normally keep you awake. Raised expression of gene ZIF-268 gene increases your memory.

Be able to describe types of pain.

Acute is self-limiting and ceases once healing occurs, chronic is persistent past healing phase and lasts 3 months or more (12 weeks).

Describe the heterogeneity of nociceptors and its functional significance.

Ad fibres - myelinated, transmit fast pricking pains: thermal and mechanical. Skin, muscles, joints. Via Lamina I and V. Allows for 1) precise localisation of stimulus 2) reflex withdrawal. C fibres - unmyelinated - transmit slow, burning pain. Via lamina I-II for peptidergic and lamina II for peptide poor. Peptidergic C fibres release peptides: Susbtance P, CGRP, which are vasoactive and make vessels more permeable - promote healing/inflammation. Thermal nociception. Peptide-poor C fibres have specific receptors such as P2X3 ATP receptors. Mechanical nociception.

Be able to describe the regions of the cortex involved in motor control

All of the frontal lobe is involved in movement. The more anterior the region, the more abstract/complex its role in movement. 1) Primary motor cortex - strip right in front of the central sulcus - Brodmann Area 4. Lesions = paralysis or paresis of specific muscle groups. 2) Premotor cortex, Brodmann Area 6 and Supplementary motor cortex, Brodmann Area 8. Damage = apraxia (inability to do complex movements) 3) Frontal eye fields - for extra ocular eye muscles. Involved in pursuit, vergence, saccades (rapid movement of eyes), emotion through facial expression. Lesion = poor perception. 4) Broca's Area - Brodmann area 44, speech production. Lesion = expressive aphasia. 5) Dorsolateral prefrontal cortex - Brodmann's Areas 9 and 10. Most complex area - does "executive functions" of planning movement from watching demonstrations, decision making, social interactions, sequencing tasks etc. This is the highest level of motor hierarchy in the brain. 6) Orbitofrontal cortex - Brodmann Area 11. Associated with the limbic system, involved in pleasure and impulsiveness. Factors like thirst, hunger and sexual appetite are regulated via the orbitofrontal cortex. Lesion = sociopathy, sexual disinhibition, impulsiveness. Parietal lobe: 1) Somatosensory cortex, directly posterior to the central sulcus - 40% of corticobulbospinal spinal tract arises from here. Brodmann Area 1, 2, 3. Motor thalamus: 1) Command centre for the basal ganglia and cerebellum - impulses refereed to corticospinal tract.

Be able to classify dementia.

Alzheimer's Disease - 62% of all cases of dementia - characterised significantly by memory loss. Frontotemporal dementia - behavioural and cognitive changes outweigh memory loss. Lewy Body dementia - alpha-synuclein deposits within neurones, glial cells and smooth muscle. Vascular dementia - in relation to insufficient blood supply such as stroke or TIA. Metabolic dementia is caused by B12 deficiency. Wernicke-Korsakoff syndrome is associated with chronic alcoholism and low levels of thiamine causing mamillary body->anterior thalamus problems. Alzheimer's symptoms: memory loss, cognitive impairments, impaired logic, confusion, agitation, aggression, depression, sleep disturbances.

Explain the neurobiology of learning.

Anatomical correlates used in learning to fear: -Long-term potentiation is the strengthening of the synapses based on recent patterns of activity. Learning of fear and anxiety occurs via long-term potentiation of the synapses of the amygdala. -Connections formed from lateral nucleus to central nucleus of amygdala. Central nucleus is main output centre --> panic, respiratory distress, sweating -Activation of lateral hypothalamus can cause spikes in HR/sweating -Activation of parabrachial nucleus can lead to panting and respiratory distress. On a neurotransmitter level, glutamate binding to NMDA receptors is responsible for fear and anxiety in conditioned responses. Blockade o these receptors lessens the dear of a conditioned stimulus. Anatomical correlates for operant conditioning: Nucleus accumbens - dopamine is rewarding/motivating.

Define the role of functional imaging in the vegetative state.

Anatomical imaging - MRIs can show injuries in particular parts of the brain eg. medial prefrontal cortex in PVS and frontal lobe injury in akinetic mutism. Functional imaging - PET scans, fMRIs and diffusion tensor imaging. The level of cerebral metabolism by % indicates whether the person is in a coma, veg state, brain death, deep sleep etc. The blood-oxygen dependent signals show you this. The default mode neurones are active when you are daydreaming/doing nothing but those in veg state have low activity in these neurones EEG - shows global slowing in electrical activity of the brain. Somatosensory evoked potentials are shocks to the median nerve - a response should be seen in the somatosensory cortex - if not, then treatments should be stopped. Event-related potential is the electrical activity seen in the brain when a stimulus is applied - visual, tactile, olfactory, gustatory and auditory. P300 is an example of ERP and is linked to the parietal lobe - it is thought to be the person having a reaction to the stimulus.

Describe how the vestibular system detects angular and linear acceleration

Angular acceleration is detected in the semicircular canals. There are hair cells which have kinetic sensitivity - they are displaced by the spinning action of the endolymph fluid in the ampullar crista. If they go one way they hyperpolarise and if they go the other way they depolarise, releasing glutamate. There is complementary bilateral signalling. When you spin at a constant rate, the endolymph fluid is moving at the same speed so there is no signal sent about acceleration. Linear acceleration (horizontal or vertical) is detected in the otoliths (saccule and utricle). Here there are calcium salt (otoconia) crystals in the gel that displace the cilia. In the utricle, the cilia are polarised towards the striola. (think of it like U, same direction). The utricle does horizontal movement. In the saccule, the cilia are polarised away from the striola. The saccule does vertical movement.

Recognise the common neurovascular syndromes and mechanisms of stroke

Anterior circulation - TACI/PACI total anterior circulation infarct/partial anterior circulation infarct (15/35%) Unilateral motor deficit Homonymous hemianopia (loss of vision of one side in both eyes) Higher cerebral function (e.g. dysphasia, neglect) Posterior circulation - POCI posterior circulation infarct (25%) Pure hemianopia Cerebellar signs Diplopia & CN palsy Bilateral/crossed sensory-motor signs Lacunar - LACI (25%) Pure motor (50%) Pure sensory (5%) Ataxic hemiparesis (10%) Sensorimotor stroke (35%)

Describe the types of amnesia.

Anterograde amnesia - cannot make short-term memories into long-term memories Retrograde amnesia - inability to retrieve long-term memories. Dissociative amnesia - blocking out critical personal memories, normally of a traumatic or stressful nature. Possible causes of amnesia - brain trauma, anoxia (stroke), infections, drug/alcohol abuse, vascular insufficiency, epilepsy, electroconvlusive therapy, degenerative diseases eg. Alzheimer's disease.

Describe the mechanism of action, pharmacokinetics and adverse effects of anticonvulsant drugs.

Anticonvulsant drugs aim to decrease the activity of sodium channels in the brain (voltage-gated sodium channel blockers): carbamazepine*, sodium valproate*, topiramate, lamotrigine*, phenytoin, lacosamide, zonisamide. Barbiturates (phenobarbitone, less used) and benzodiazepines (clonazepam* for sedation) block GABA-A receptors. Drugs that work at calcium channels (calcium channel blockers) include ethosuximide* (petit mal) and gabapentin/pregabalin (target a2d subunit of calcium channels). Perampanel blocks AMPA glutamate receptors. Felbamate blocks NMDA glutamate receptors. Levetiracetam targets ntm release (NTM release inhibitor). Tiagabine inhibits GAT-1 ntm reuptake. Vigabatrin prevents GABA synthesis by inhibiting GABA transaminase (NTM synthesis inhibitor). Most commonly used are marked with a *. Notable problem with anticonvulsants is their toxicity to unborn child - teratogenic. Alternatives - DBS, surgery (resection of affected lobe, lesioning corpus callosum), vagal nerve stimulation, ketogenic diets.

What are the components of the MSE : mental state examination? (Not to be confused with MMSE which tests for dementia by looking at memory)

Appearance, behaviour, rapport, speech, mood, thought.

Be able to discuss new therapeutic approaches in Alzheimer's Disease.

B-secretase inhibitors A-beta oligomer vaccines and monoclonal antibodies Tau lowering/anti-aggregation compounds (CSP-1103) Notch-sparing y-secretase inhibitors or modulators. A-beta aggregation inhibitors. Regulation of abnormal anti-inflammatory mechanisms.

Provide examples of psychological treatments based on theories of learning.

Befriending people with mental health illness. Teaching psychiatric patients social skills. Using token-economy systems in institutions - people who display desired behaviours are given tokens to exchange for desirable items. Reward for punctuality.

Know some anxiolytic drugs, their mechanisms of action and side-effects

Benzodiazepines (clonazepam, alprazolam...) 5-HT1A agonists (buspirone, ipsapirone) partial agonists β-adrenoceptor antagonists (propranolol...) SSRIs (fluoxetine, escitalopram, paroxetine...) Barbiturates (obsolete)

Describe the mechanisms triggering and maintaining sleep.

Blood chemicals can promote/inhibit sleep as they are detected by the medial and ventrolateral preoptic nuclei. The medial and ventrolateral preoptic nuclei have projections to the tuberomamillary nucleus (TMN) - in the TMN are histaminergic neurones which are active in wake and silent in sleep. Histamines generally keep you awake --> antihistamines therefore make you drowsy. CCK, adenosine, melatonin and GABA send you to sleep. Ghrelin and caffeine have arousal effects - caffeine is an antagonist for A1 adenosine receptors and keeps you awake. Daylight/diurnal rhythms: Photoreceptor cells in the retina continually fire in daylight --> impulses referred to the suprachiasmatic nucleus of the hypothalamus --> impulses referred to the reticular formation of the midbrain --> keep you awake. Lesions of the suprachiasmatic nucleus destroy normal sleep cycles in animal models - do sleep but irregulary.

Introduce mechanisms underlying learning and memory (LTP).

Brain can re-wires itself by creating new collaterals or increasing the number of synapses between neurones to create memories. Long-term potentiation is when there is a persistent strengthening of synapses based on recent patterns of activity. It results in long-lasting increases in signal transmission between two neurones. Ligand-gated/ionotropic NMDA receptors in hippocampus have been shown to play important role in memory. When blocked, rats could not find the platform in a water maze they had previously been able to locate. The NMDA receptors is most important for memory eg. GRIN 1, 2A, 2B. (NMDA receptor types to know NMDA, AMPA, kainate). NMDA receptor is targeted by memantine. It is thought that by blocking the "noise" at these receptors other impulses can get through. Magnesium has been shown to block NMDA/AMPA receptors - the receptors cannot conduct when the membrane potential of a neurone is negative. Plasticity in the brain: synapse number, vesicle number, AMPA receptor number and AMPA receptor state (phosphorylated?) are factors that determine the plasticity of the brain.

Understand how chemicals within cannabis can control symptoms of multiple sclerosis.

CB1 receptor is the most abundant in the body (cannabinoid 1). THC (tetrahydrocannabinol) enters body cells via the CB1 receptor. THC directly inhibits voltage-gated calcium and sodium channels. Overall effect: signal transduction pathways may be activated or inhibited depending on where in the brain they are. In MS, CB1 receptor stimulation can lower spasticity and tremors. Sativex is a spray with THC and cannabionoidal which opens K+ channels to promote K+ efflux and keeps the inside of the cell more negative. The CB2 receptor is how endogenous cannabinoids enter cells - anandamide and 2-AG (arachiodonyl-glycerol). Fatty acid amide hydrolase is the enzyme responsible for the breakdown of this. *Antispastic drugs used in MS: gabapentin, GABAa agonists, benzodiazepines, baclofen, nerve blocking agents e.g. botulinum toxin A, dantrolene. Benzodiazepines such as diazepam open Cl- channels to promote influx of Cl- to keep the membrane potential negative.

List some of the biomarkers used to detect Alzheimer's Disease.

CSF markers: AB40 AB42 in CSF - decreased from normal values. Phospho-tau 181 and 231 in CSF - increased from normal values. Bloods - protein, DNA/RNA, lipids MRI Scan - changes in brain structure, particularly atrophy in the temporal and neocortex regions. PET Scan using fluorine 18: C11 Pittsburgh compound B (AB plaques) Fluorine 18 fluorodeoxyglucose (FDG) 18F-tagged tau 807 and 808 ligands.

What structures are included in the basal ganglia?

Caudate, putamen, globus pallidus (interna and externa). The basal ganglia also has functional motor connection to the subthalamic nucleus in the ventral thalamus and substantia nigra in the base of the brainstem. The corpus striatum is made of the caudate, putamen and globus pallidus.

Be able to describe alterations in cholinergic neurotransmission in Alzheimer's Disease.

Cholinergic transmission decreases in the cortical regions/limbic systems. Thus acetylcholinesterase inhibitors are used - they offer modest improvement in symptoms.

Discuss the types of chronic pain/pain insensitivity and treatments.

Chronic pain - pain that lasts for greater than 12 weeks (3 months). If it is nociceptive (due to actual tissue damage) it can be treated with NSAIDs, opiates, steroidal anti-inflammatories. If the pain is maladaptive it persists past the healing phase and signifies abnormal activity in the neural system. Maladaptive pain is divided into neuropathic or dysfunctional. Neuropathic - after an injury when there is damage to CNS/PNS. Dysfunctional is spontaneous characterised by hyperalgesia and allodynia. Causes of maladaptive pain include strokes, infections, diabetes, "partial injuries, chronic alcoholism. Maladaptive pain is treated with anticonvulsants and antidepressants eg. amitriptyline. Alternative: acupuncture dampens pain via A-beta or A-delta fibre activation, - stimulates periaqueductal grey --> DNIC of pain "diffuse noxious inhibitory control". An inhibitory interneurone releasing GABA/ENK/5-HT dampens the input. CIPA - congenital insensitivity to pain with anhydrosis - a loss of C fibres due to a NGF (nerve growth factor) mutation. Patients cannot feel pain, cold, heat. Can result in pressure sores, loss of blood supply --> tissue infarct --> gangrene and infections. Diabetic neuropathy happens when fine nociceptors of the periphery are damaged by prolonged hyperglycaemia - common presentation is diabetic foot ulcer. Loss of transudction or transmission is due to faulty sodium channels (defective subunit NaV1.7) so action potentials cannot be propagated properly. Congenital indifference to pain.

Where does pain come from and how it explains morbidity and mortality.

Chronic pain is associated with major depressive disorder/mood/anxiety disorder and addiction. These conditions are though to overlap in regulation and can possibly aggravate one another. Same neurotransmitters/lack of are responsible for depression and chronic pain: dopamine, serotonin, opioid system, noradrenaline. Opioid system and dopamine dampens pain. Depression and anxiety disorders are associated with suicide risks of 12 and 7. Pain and disability are independent factors for suicide.

Be able to list the major classes of drugs that are subject to abuse

Class A - heroin, methadone, cocaine, crack cocaine, LSD/ecstasy, magic mushrooms Class B - cannabis, amphetamines, ketamine, barbiturates, codeine, methylphenidate Class C - anabolic steroids, benzodiazepines, khat, gamma hydroxybutyrate, gamma butyrolactone.

To explain the principles of learning and conditioning as applied to addiction

Classical conditioning - the learning of an association. When two stimuli are repeatedly paired they can elicit the same response. Eg. alcohol taste makes you feel good. Brain regions involved in this are the nucleus accumbens and cerebellum? Operant conditioning - positive reinforcement and negative reinforcement. Positive reinforcement - behaviour leading to pleasurable sensation is likely to be repeated eg. sensation, satisfaction of biological needs, social reinforcement, group membership. Negative reinforcement - idea that you can avoid negative sensations by doing certain actions: a drug can reduction in habitual stress, reduction in acute distress, reduction of withdrawal symptoms.

Describe the role of learning in addictive behaviours.

Classical conditioning in alcoholism - do not like taste of alcohol at first (unconditioned stimulus), but the pleasant feeling of being drunk (unconditioned response) can later cause the taste of alcohol to become pleasant (conditioned stimulus/response). Garcia effect - when you avoid something because of past unpleasant experiences with it. Eg. food poisoning from sushi means you never have sushi again. Furthermore, stimulus generalisation means you might not eat fish any more. Classical conditioning is used in advertising - conditioned emotional responses means that you associate sex/positive feelings with a certain product. Eg. a particular wine advert will show you a sexy holiday with good-looking partner.

How do some drugs alter neurones?

Cocaine causes increased spine density and neuronal branching of the prefrontal cortex and nucleus accumbens medium spiny neurones. Morphine causes the opposite - deceased spine density and neuronal branching.

To see the concept of addiction in its social context.

Concept of addiction changes with time in history, geographical location and social grouping. The 21st century version of addiction differs from that of 100 years ago.

Define terms consciousness, coma and sleep

Consciousness - a state of full awareness of one's surroundings and oneself Coma - a state of unarousable unresponsiveness with or without reflexes. Sleep - a natural periodic state of rest for the mind and body during which consciousness is partially or completely lost so that voluntary movements and postural reflexes also stop. Consciousness in terms of electrical activity - appears to be an asynchronous pattern of firing in the cerebral cortex. Lends plausibility to the idea that neuronal programming will increased the level of consciousness and can be done by puzzle solving. Consciousness is all about the pattern of firing, not metabolic activity. Assessment of consciousness - the ability of an individual to react appropriately to stimuli in the outside world (assessed by Glasgow coma scale)

Define what consciousness is and its components.

Consciousness is a state of full awareness of oneself and one's surroundings. The two components of consciousness are arousal and content. Or wakefulness and awareness.

Describe the anatomical correlates of consciousness

Consciousness is divided into arousal and content. Arousal refers to sleep and wake cycles, whereas content refers to the processing/interactions with the surroundings and events. Anatomical correlates of arousal: the reticular activating system. Raphe, pedunculopontine, locus coeruleus and ventral tegmental nuclei. Anatomical correlates of REM sleep: laterodorsal tegmental and pedunculopontine nuclei of the midbrain - project cholinergic axons to the thalamus. Anatomical correlates of the vegetative state: the mesocircuit model 1) Prefrontal cortex on medial side 2) Parietal/occipital/temporal lobe 3) Central thalamus What happens: the medial side of the prefrontal cortex and the parietal cortex are lesioned/damaged causing a disconnect between the frontal and parietal lobes. The thalamus is thought to be the relay point between the two and may also be lesioned and damaged.

Understand the principles of management of patients who have suffered a stroke.

Consider Thrombolysis* Transfer to Stroke Unit Review antiplatelet therapy* MDT Rehabilitation Nursing: Positioning, bedsore management Speech and Language Address risk factors Diabetic Control Antihypertensives Statins Cardiac disease Carotid disease Treat Complications Nutrition Cerebral Oedema Infection & pressure sores DVT Seizures & Spasticity Depression Physiotherapy to avoid Therapy OT Sitting balance, walking aids, transfers, adaptations, home visits contractures Advice & Education Driving Contraception Work Weight-loss Exercise Smoking Benefits

Review the central structures involved in the primary headache.

Cortical spreading depression: A transient and local suppression of spontaneous electrical activity in the cortex, which spread slowly across the brain. There are abnormal glutamate, calcium and magnesium concentrations in the brain in cortical spreading depression. Cortical spreading depression causes meningeal arteries to swell up and release substance P, neurokinin A, CGRP (calcitonin-gene-related peptide). This is neurogenic inflammation. Local nerves are activated, relay the pain to the trigeminal ganglion. Peripheral sensitisation takes place at V1 dermatome (forehead/ophthalmic region) causing pain around/behind the eyes. Impulses are relayed to the trigeminal nucleus caudalis in brainstem via trigeminal nerve. Central sensitisation takes place - allodynia due to neuronal hyperexcitability - can last up to 10 hours. These are the five stages for a migraine: 1) Premonition - craving sweet foods, fluid retention, heightened perception, tiredness, neck pain (begins in hypothalamus) 2) Aura - visual normally, can have feeling in tummy (caused by cortical spreading depression) 3) Physical manifestation of migraine - headache with vomiting, nausea, malaise, lethargy, poor concentration, difficulty focussing, sensitivity to light, sound and smells 4) Resolution - vomiting/deep sleep/medication 5) Recovery - feel hungover, diuresis, lethargy, limited food tolerance

Describe and classify anxiety disorders

DSM-V can be used to diagnose anxiety disorders. 1) Anxiety disorders include: Panic disorder Agoraphobia Social phobia (social anxiety disorder) Simple phobia Post-traumatic stress disorder Generalized anxiety disorder 2) Obsessive-Compulsive and Related Disorders: Obsessive compulsive disorder Trichotillomania Hoading disorder

To be able to define declarative memory and procedural memory.

Declarative memory is explicit factual data. It is divided into episodic and semantic. Episodic is past events eg. where was your last birthday party. Somatic refers to learnt knowledge eg. current prime minister and concepts from school. Procedural memory (aka non-declarative) is implicit - consists of skills and habits such as an ability to play tennis or chop vegetables.

List some non-pharmacological treatments of depression.

Deep brain stimulation of subgenual cingulate cortex. Electroconvulsive treatment for resistant depression/continued suicidal ideation/uncontrolled bipolar disorder. Cognitive behavioural therapy

Give specific examples of central nervous system depressants, stimulates and other types of drugs associated with dependence.

Depressants - benzodiazepines, alcohol, narcotics, barbiturates, marijuana Stimulants - caffeine, amphetamines (ecstasy is a substituted amphetamine), cocaine

Understand the direct and indirect pathway and its contribution to movement

Direct: Substantia nigra (excitatory dopamine via D1) and sensory association cortex (excitatory Glutamate via NMDA/AMPA) excite the striatum. Striatum sends inhibitory via GABA impulse to the globus pallidus interna --> sends inhibitory impulse via GABA to the motor thalamus. Disinhibition --> excitation of the motor thalamus --> activates supplementary motor cortex via glutamate. Indirect: Substantia nigra (excitatory dopamine via D1) and sensory association cortex (excitatory Glutamate via NMDA/AMPA) excite the striatum. Striatum sends inhibitory via GABA impulse to the globus pallidus externa --> sends inhibitory impulse via GABA to the subthalamic nucleus. Subthalamic nucleus sends excitatory signal via glutamate to the globus pallidus interna --> sends inhibitory signal to the motor thalamus --> no change in motor programme.

Be able to describe the main types of acetylcholinesterase inhibitors and their use in dementia.

Donepezil, rivastigmine, galantamine. They provide symptomatic relief as cholinergic neurones are depleted in Alzheimer's Disease. Acetylcholinesterase normally breaks down ACh into acetate and choline in the synaptic cleft but this is prevented. therefore AC ha a longer duration of action in the synaptic cleft --> stabilise/lessens the cognitive issues of Alzheimer's. To be used in mild to moderate Alzheimer's Disease.

Describe the treatment of Huntington's disease.

Dopamine antagonist/neuroleptics - haloperidol, olanzapine, tetrabenzadine - deplete vesicles of dopamine (lower the direct pathway activity). Atypical antipsychotics, tricyclic antidepressants - amitriptyline, imipramine.

Review the circuitry of the basal ganglia, its main neurotransmitter and its pathological modifications in Parkinson's Disease/Huntington's Disease.

Dopamine via D1r is excitatory in direct pathway. Dopamine via D2r is inhibitory in indirect pathway. Striatum has GABAergic inhibitory neurones. Subthalamic nucleus has Glutamic excitatory neurones. In Parkinson's there is loss of dopamine from substantia nigra pars compacta - the direct pathway becomes less active and person finds it harder to change motor programmes. There is much more of the indirect pathway - an imbalance. In Huntington's, there is a loss of GABAergic neurones of the striatum, much more of the direct pathway takes place - so there is increased drive of motor thalamus.

How is pain managed psychopharmalogically and what adjunctive therapies are available?

Drug abuse/addiction causes hit of dopamine every time you take the drug - the neurotransmitter balances in the brain change. Need antidepressants, antipsychotics and mood stabilisers to work on serotonin, dopamine and nordadrenaline systems of the brain. Adjunctive: CBT, counselling, hypnosis, distractions, biofeedback, relaxation therapy, acupuncture. Typical pain management programme: 1) Aggressive pharmacotherapy - titrated drug 2) Weaning off opiates 3) Cognitive behavioural therapy, 12-week course 4) Follow up, drug dosage changed.

Define drug dependence, drug tolerance and drug abuse

Drug dependence - an adaptive state in which repeated drug use results in withdrawal symptoms on cessation of usage. Drug tolerance -the diminishing effect of a drug which comes with repeated use. Drug abuse - a pattern of drug use that poses risks either directly or indirectly to the individual using the drug or others.

Know some of the major unresolved issues with depression and treatments.

Drugs need to be taken for 12 weeks acutely, then continued for 6 months after the remission due to... Antidepressant drug discontinuation syndrome - a set of symptoms caused by sudden cessation/sharp tapering off of the drug. Must be tapered gradually over many months to prevent this. Symptoms include - anxiety, insomnia, headaches, nausea, electric shock sensations, agitation, mood swings, abdominal discomfort. (Kind of like withdrawal symptoms) Delayed onset of antidepressant drugs - suggests that there is not simply problem with synaptic concentrations of amines but complex neurosignalling pathways that are altered and regulatory adaptations that need to be adjusted. Suggested mechanism: Changes in autoreceptor action at the synapse - initially an overstimulation of autoreceptors decreases firing of action potentials in a presynaptic membrane, however, after a week or two the auto receptors desensitise to the higher concentrations and firing activity returns to normal. Depression is underdiagnosed and undertreated. 60-70% of patients do not go into remission even with drug treatment. The drugs have side effects, delayed onset, must be continued 6 months after remission, antidepressant drug discontinuation syndrome.

Describe the role of genetics in Alzheimer's Disease.

Early-onset Alzheimer's is linked to: -Presenilin 1 and 2 genes (PSEN1 and PSEN2) on chromosome 14 and 1 respectively. They are both coding for subunits of y-secretase. PSEN1 is the most common. -APP gene found on chromosome 21 - explains link with Down Syndrome. Late-onset Alzheimer's is linked to: -ApoE4 on chromosome 19. Mutations in MAPT (microtubule-associated protein tau) are not associated with Alzheimer's but are associated with frontotemporal dementia and other tauopathies.

Describe the pathophysiology of epilepsy.

Epilepsy: a chronic neurological disorder characterised by unprovoked seizures. It has neurobiological, psychological, social and cognitive implications. Epileptogenesis is the process by which normal brain function changes towards generation of abnormal electrical activity. Structural alteration in epilepsy: 1) In temporal lobe epilepsy, CA2 and CA3 regions of the hippocampus are lost/degenerate. In these sclerotic regions there is mossy sprouting of granule cells which can create reverberating excitatory circuits - reshaping the brain. Neurogenesis may occur and create aberrant circuits. 2) Loss of GABAergic chandelier cells. Normally these synapse on the axon initial segment of pyramidal cells in the cortex and is inhibitory. Loss of this therefore causes excitatory input to pyramidal cells and excessive activity of corticospinal/pyramidal tract Other: previous brain infection/injury such as viral encephalitis, brain tumours, TBIs, strokes, aneurysm.

To name the brain areas associated with acquisition and storage of declarative/procedural memory.

Episodic memory - hippocampus, medial temporal lobe, neocortex Semantic memory - anterior/lateral temporal cortex, prefrontal cortex Procedural memory - motor cortex, cerebellum and striatum

What is the neurochemical imbalance in schizophrenia?

Excessive activation of D2 receptors in the mesolimbic pathway (ventral tegmental area to the nucleus accumbens). It has also been noticed that there are decreased numbers of NMDA glutamate receptors in schizophrenic patients. This is being considered as a new drug target.

Symptoms of the mind: mania

Flight of thoughts, pressure of speech, increased speed of speech, elation "high" mood, energised, hyperactive behaviour, disinhibition, inappropriate sexual advances, gambling or spending, grandiosity, reduced sleep

Describe the present treatment of multiple sclerosis.

Four categories of treatment: 1) Neuroprotective- beta-interferon 1a and 1b, intravenous. Mechanisms of action: prevent T-lymphocytes passing BBB, suppresses T-lymphocyte activity, lowers gamma-interferon/toxin production, increase T2 suppressor cell number to dampen immune activity. 2) Immunological - monoclonal antibodies: Natalizumab targets alpha-4 integrin, a cellular adhesion molecule, to stop T-lymphocytes getting through BBB. Alemtuzumab* - targets CD52 receptors of B/T-lymphocytes. Ocrelizumab - targets CD20 on B cells. Daclizumab - a monthly injection that blocks IL-2 receptors, destroys the lymphocytes/macrophages destroying myelin in body. Fingolimod (FTY720) - orally administered novel immunosuppressant that traps lymphocytes in lymph nodes by acting on 5-sphingosine-1-phosphate receptors. Teriflunomide is an immunological modulator. 3) Remyelinating - Glatiramer acetate (Copaxone) resembles one of the proteins involved in making myelin, blocks T cells from attacking myelin, switches off T1 attacking lymphocytes and increases T2 lymphocytes (immune suppressor) activity 4) Regrowth of neurones -HSCT* - haematopoietic stem cell transplant Baclofen - CNS depressant agonist of GABA receptors has good effects against spasticity. Mitoxantrone* - a chemo agent not licensed for MS but used on individual basis Cladribine - another anti-cancer drug, slows relapse rate by 58% - depletes T/B cells in body. Drugs with * are induction therapies. Otherwise are maintenance-escalation therapy.

Describe the microanatomical arrangement of the cerebellum including the cortical morphology and connections of the deep nuclei.

Four deep cerebellar nuclei that modulate all the inputs and outputs from the cerebellum: FGED Fastigial Globose Emboliform Dentate These are from medial to lateral (there are two sets - one set on either side of vermis, with fastigial being in the vermis) Three layers: 1) outer molecular layer - pale, not very cellular 2) middle layer - single layer of purkinje cells 3) inner layer - thick granule layer, lots of granule cells/granule cell bodies.

Describe the functional subdivisions of the cerebellum and the effect of injury in different regions.

General about cerebellum: smooths out movements, creates accurate movements. modulates and refines the commands from the primary motor cortex using proprioception and other sensory organs. General cerebellar injury causes clumsiness, abnormal fatigue, instability of movement. Extraocular eye muscles affected. The functional subdivision of the cerebellum: 1) Vestibulocerebellum - works with the vestibulospinal tracts to coordinate head and eye movement and maintain stability of gaze/balance and posture. The motor programmes for these movements are stored in the flocculonodular lobe. 2) Spinocerebellum - Contains the anterior lobe of the cerebellum - control locomotion and limb co-ordination. Interposed nuclei - emboliform/dentate and fastigial (as spinal cord attaches via vermis) are part of spinocerebellum. Outputs are lateral vestibulospinal tract and reticulospinal tract (not CST). 3) Cerebrocerebellum - Contains posterior lobe (cortices are connected via dentate). Modulates and smooths out motor commands from the primary motor cortex. Output through superior cerebellar peduncle --> ventral thalamus. Injury: 1) Flocculonodular syndrome - poor balance, disordered eye movements, nystagmus, poor visual tracking, cannot perform Romberg test, truncal ataxia (cannot balance while sitting on a bed), nodding of head. Main cause of vestibulocerebellar problems is medulloblastoma in the 4th ventricle - this is where flocculonodular lobe is. 2) Anterior lobe syndrome - incoordination of limbs, ataxia, hypotonia, dysdiachokinesis, depressed/pendular reflexes. Often seen in alcoholics due to lack of B vitamins. 3) Neocerebellar syndrome - loss of hand-eye co-ordination, intention tremor, dysdiachokinesis, dysmetria, poor articulation/slurring, deficits in selection and attention. *Alcohol intoxication mimics some of these - slurred speech, ataxic gait, double vision, inco-ordinaton, intention tremor etc. Because cerebellum has many GABAergic neurones sensitive to alcohol.

Understand the pathological changes which occur in the brain following head injury.

General axonal response to injury: 1) After injury, axons separate and form retraction balls 2) Damaged neurones undergo apoptosis 3) Neurones swell as cellular metabolism is disrupted. Injured cells lose energy and ion metabolism and swell causing cytotoxic oedema. Haematomas: Extradural/epidural are smooth shape on scan - high pressure. Normally it is where there is a skull fracture which has pierced middle meningeal artery. Subdural haematoma - arachnoid splits from dura and fills with blood - sickle shape. Subarachnoid haemorrhage follows the sulci-gyri of the brain fissures. Cerebral contusion is brushing the brain at the layer of pia mater. Intracerebral hematomas are bleeds into brain tissue. Duret haemorrhages - small lineal areas of bleeding in the midbrain and upper pons of the brainstem. Caused by traumatic downward displacement of the brainstem.

Review the neurobiology of schizophrenia

Genes identified: -Dysbindin -highly indicated as a genetic component - may affect dopamine levels at D2 receptors/ GABA and glutamate transmission. Found on chromosome 6. -COMT - involved in NTM synthesis and breakdown of dopamine/5-HT chr.22 -DAOA - chr.13 -BDNF - chr.11 -Neuregulin 1 - chr.8 - involved in neuroplasticity -DISC1 - chr.1 - involved in neurodevelopment and signalling in corticolimbic areas Structural alterations to the brain: -enlargement of the ventricles -smaller mesial temporal lobe structures -blood for to the prefrontal cortex decreases (hence executive function deficits/cognitive issues) "hypofrontality" -decreased number of synaptic spines and dendritic complexes in neurones (due to abnormal neurodevelopment) -default mode network more active in schizophrenic patients

Why do some people get addicted and others don't?

Genetic do play a part in addiction - some people will have susceptibility genes to opiate (depressant)/cocaine (stimulant) addiction: OPRM1, OPRD1, OPRK1, PDYN, PENK which all work in opioid system. DBH, DRD2, SLC6A3 all work on the dopaminergic system. SCL6A2 works on the noradrenergic system. SCL6A4 works on the serotonerhi.c The fewer pleasures receptors you gave (dopamine) the more susceptible you will be to drug addiction as the easier you will find it to get high.

Describe the genetics of depression.

Genetic risk factors: 5-HT transporter polymorphism Carriers of MAOA-H (higher active MAOA risk allele)

Describe the genetics of epilepsy.

Genetics of epilepsy is complex - many ion channel genes are linked to it "channelopathy". SCN1B (sodium channel) gene mutations can cause dysfunction in the sodium channels of the brain - large influxes of Na+ --> increased excitability of the neurones and brain causing febrile seizures. Similarly, KCNQ2 (potassium channel) gene mutations can cause K+ channels to stay open --> rapid efflux of K+ --> increased excitability of neurones causing benign familial neonatal convulsions (recurrent seizures in newborns). Others: SCN2A, GABRA1

Be able to describe the main types of epileptic seizure.

Grand mal - "big" obvious seizures: The tonic-clonic seizures. 1) Premonition/aura - sensation or smell 2) Pre-tonic-clonic phase - small myoclonic jerks 3) Tonic phase - epileptic cry, fall to floor, tonic contraction of the limbs, eyes roll back, pupils dilate, cyanosis 4) Clonic phase - massive repeating jerks of increasing amplitude followed by relaxation (sphincters may open) 5) Postictal phase - generalised fatigue, muscle soreness, decrease muscle tone, headache, needs reassurance. Petit mal - The absence seizures: short stares or absences of 5-7 seconds "blackouts". Jacksonian seizures - focal motor seizures that start at thumb/big toe and move to other parts of the body. Akinetic type epilepsy - fall, stumble, drop what you are holding Myoclonic type - muscles contract quickly and repetitively. Other: olfactory hallucination, disrobing, peculiar behaviours, memory changes, feeling something rising in the stomach, deja vu

Describe the pathophysiology of stroke in relation to risk factors.

Haemorrhagic - Bleed until it tamponades. Hypertension is the primary cause of haemorrhagic stroke, Charcot-Bouchard microaneurysms rupture. Can also be caused by weakened walls which leak. Secondary effects include herniation, raised intracranial pressure (compromises global brain perfusion). Can be also caused by tumours, trauma, coagulopathy. Ischaemic - Risk factors are arteriolarsclerosis, atherosclerosis, cardiac embolisms from atrial fibrillation. Lacunar stroke (small vessel). The ischaemic cascade results in cellular swelling and lysis by excitotoxicity. One area of brain infarcts rapidly - the infarct core in the centre, around which there is brain tissue critically perfused, electrically inactive then a penumbra of tissue which can be restored but is at risk. Other risk factors: hypertension, hypercholesterolaemia, diabetes, vascular disease.

Describe tests for balance disorders

Hallpike manoeuvre tests for BPPV (benign paroxysmal positional vertigo) - turn patient's head to one side as you lower them to a table. If there is nystagmus, the ear facing the floor is affected. Caloric reflex test - drop cold/warm water into the ear. Cold = eyes to ipsilateral side and horizontal nystagmus. Warm = opposite. Videonystagmography - using infrared goggles to trace eye movements. Electronystography Posturography, Bárany chair - put someone in a spinning chair and then get them to point at a card.

Discuss spatial memory.

Hippocampus is involved in spatial memory - is enlarged in taxi drivers for example. It is thought that the hippocampus has maps but also is used in planning routes - "place-cells" here.

Be able to describe the effects of lesions in the amygdala and fornix

Hippocampus is the librarian who labels memories with location, time and other parameters eg. who you were with, but it is not the memory store itself. Lesion to hippocampus --> anterograde amnesia - cannot transfer short-term memories to long-term. Actual store - parahippocampal structures. Amygdala labels memories with emotional quality. Lesions to amygdala --> patients find it difficult to identify emotions through facial expressions, misinterpret frightening experience and may take risks, have odd relations and/or make poor financial decisions. Overactivity of the mesolimbic system causes schizophrenia, underactivity causes Parkinson's. It is thought that dopamine deficient depression is caused by underactivity in the mesolimbic pathway also. Blocking dopamine receptors in the nucleus accumbens may be the answer to preventing addiction.

Describe the pathophysiology, clinical symptoms and basic treatments of movement disorders.

Huntington's and Parkinson's - see learning objectives for Pharm of movement disorders.

Understand the treatment ladder to maintain no evidence of disease activity: NEDA

In an attempt to achieve NEDA, people start a disease-modifying drug as soon as they receive their diagnosis. The aim is to achieve a state where the patient has -no relapses -no increase in disability -no new or active (enhancing) lesions on their MRI scans.

Understand the role of thrombolysis in the treatment of acute ischaemic stroke

Is the current best treatment for stroke. No change in mortality. Thrombolytic drugs currently available: streptokinase, tissue plasminogen activator

Explain the pain triumvirate.

It explains the three main networks of the brain involved in pain. 1) "Sensory/discriminative" Lateral thalamic system pain intensity/location 2) "Emotional/motivational" ACC, medial thalamic system, basal ganglia, insula, amygdala register unpleasantness of pain. Reward from escaping pain. 3) "Cognitive/evaluative" interprets meaning of pain - ACC and dorsolateral prefrontal cortex.

Be able to describe Kluver-Bucy syndrome, temporal lobe epilepsy and Wernicke-Korsakoff syndrome.

Kluver-Bucy Syndrome - Seen in rhesus monkeys in 20th C experiment. Removed the amygdala - monkeys lost all sense of fear, and stopped socially interacting with one another. symptoms: psychic blindness, oral tendencies, hypermetamorphorsis (appropriate behaviour at inappropriate time), altered sexual behaviour, emotional changes. Temporal lobe epilepsy - occurs when there is a spread of depolarisation in the temporal lobe, hippocampus and amygdala of the brain. Produces seizures of strong emotions eg. fear or intense joy, can also involve smells, tastes, touches, hallucinatory state. Possibly preceded by aura/deja vu. Sometimes due to labour which squeezes baby's temporal lobes. Wernicke-Korsakoff Syndrome - A type of encephalopathy associated with chronic alcoholism - linked to deficiency in thiamine vitamin B1. Causes anterograde amnesia, retrograde amnesia, confabulation and apathy. It is thought that the mamillary body's connection to the anterior thalamus is disrupted. Treat with thiamine supplements for 2-3 days.

Distinguish different types of conditioning and learning.

Learning: Deliberate and informal. Auditory, kinaesthetic, visual. Conditioning: Classical conditioning "Pavlovian" - A stimulus acquires the capacity to evoke a reflexive response originally elicited by another stimulus. Phases: acquisition, extinction, spontaneous recovery. Conditioned/unconditioned stimulus/response. Stimulus generalisation - being scared of all animals when only one bit you. Stimulus discrimination - being scared of only mice. Operant conditioning divides into positive and negative reinforcement. Works by reward and punishment - Law of Effect (human behaviour is random, some behaviours are rewarding and will be repeated and those that cause punishment will not be). Principles of reinforcement: fixed ratio, fixed interval, variable ratio, variable interval. Punishments: positive punishments are getting bad grade, being hit, being laughed at. negative punishments are removing pleasurable stimuli, taking away toys, grounding your kids *Not all behaviours are learned, some are there to protect us like orienting reflexes, habituation, dishabituation, instincts, imprinting.

Appreciate the effect of pathological processes of this system on conscious level

Lesions to the suprachiasmatic nucleus stop normal diurnal rhythms of wakefulness from occurring - sleep happens irregularly. Loss of orexins in the brain causes narcolepsy. Drugs can affect the sleep cycle: -Amphetamines and cocaine cause release of dopamine and noradrenaline even when monoaminergic signals from the reticular activating system are not there. -SSRIs interfere with REM by maintaining serotonin concentrations in the brain - increased sleep onset latency, waking often, dream changes. -Tricyclic antidepressants eg. Amitriptyline promote sleep by blocking H1 histamine receptors and blocking noradrenaline reuptake. -Monoamine oxidase inhibitor tranylcypromine is structurally similar to amphetamine and causes sleep disturbances -Monoamine oxidase inhibitor phenelzine is sedating but may suppress REM sleep.

Be able to describe the limbic structures - the hippocampus, the amygdaloid and septal nuclei.

Limbic system aids survival by creating emotions, motivations and memory. It is connected to the olfactory system and reward circuits of the brain. Four cortical regions of the limbic system: 1) Orbitofrontal cortex - response to threat/danger 2) Anterior cingulate cortex - rostral part does response to pain/caudal part does quality of pain 3) Posterior cingulate cortex - visuospatial memory, time and space 4) Parahippocampal cortex - learning and memory All supplied by anterior cerebral artery/posterior cerebral artery. Subcortical regions of the limbic system: 1) Hippocampus - curves along medial side of inferior horn of the ventricles. Output fibres come out of its fornix, hanging off the septum pellucidum. Reaches into the hypothalamus where there are septal nuclei and mamillary body of nucleus. 2) Amygdala - involved in panic, fear and terror. Embedded in the anterior medial temporal lobe in the "entorhinal cortex". It is made of many small subnuclei The ventral striatum of the limbic system: 1) Nucleus accumbens - investigate and terminate behaviours which bring about reward eg. eating. Belongs to mesolithic system: dopaminergic projections from ventral tegmental area of midbrain. 2) Septal nucleus 3) Nucleus basalis (nucleus of Meynert/basal nucleus) - ACh. *Remember that the dorsal striatum is made up of the caudate and putamen, involved in planning/initiating the next motor activity, whereas the ventral striatum is involved in motor actions using reward/punishment motivation, also uses dopamine.

Describe local and general anaesthetics.

Local anaesthetics block sodium channels. Can be administered to skin, via epidurals. SFX: bradycardia, respiratory depression. Examples: lignocaine, bupivacaine, prilocaine General anaesthetics: halothane, nitrous oxide, xenon, enflurane, isoflurane, propothol, thiopental, ketamine. Both excitatory and inhibitory action. Excite inhibitory receptors and inhibit excitatory receptors. Administered intravenously or inhaled. May induce cardiovascular depression.

Understand the disease processes in multiple sclerosis

MS is a chronic inflammatory, neurodegnerative disease of the myelin in the brain. Demyelination is the key pathology due to macrophages engulfing myelin at the site of multifocal lesions. There is also oligodendrocyte loss. Most often characterised by relapsing-remitting presentation. Within 25 years ambulatory problems will occur.

Give an overview of the anatomical correlates of sleep and wake.

Main anatomical correlate for sleep/wake is reticular activating system of midbrain and rostral pons - has diffuse neuronal connections to the cerebral cortex. Within the reticular activating system from more rostral to more caudal: ventral tegmental nucleus has dopaminergic neurones, locus coeruleus has noradrenergic neurones, raphe nucleus has serotonergic neurones, pedunculopontine nucleus has acetylcholinergic neurones. These nuclei are are "engines of wakefulness". Hypothalamic structures eg. the suprachiasmatic nucleus, the medial and ventrolateral preoptic nuclei, orexigenic neurone cell bodies in the posterior hypothalamus and tuberomamilary nucleus. *know that the nucleus basalis goes to the cerebral cortex while the pedunculopontine nucleus goes to the thalamus.

Be able to describe the functional characteristics of the major motor tracts (pyramidal, reticulospinal, vestibulospinal) and their origins

Major pyramidal tracts: 1) Corticobulbospinal tract. Arises from the somatosensory cortex (Brodmann Areas 1, 2, 3). Feeds through internal capsule where it is susceptible to damage from stroke. Divides into... 2) Corticospinal tract - arises from pyramidal cells of the primary motor cortex (premotor and somatosensory nerves feed in a bit too). Decussates at the pyramids C1-C5. Has two parts: Lateral corticospinal tract which does movements of limbs and digits (90%). Medial corticospinal tract (10%) does movement of the trunk - this decussates in the cervical region and you will not find its neurones below this point. 3) Corticobulbar tract - terminates on cranial nerve nuclei, pontine nuclei, red nucleus and reticular formation to control movements in the head and neck. Extra-pyramidal tracts: 1) Medial vestibulospinal tract - minor extra-pyramidal tract, role in moving head to tract objects with eyes. Continuation of medial longitudinal fasciculus. 2) Lateral vestibulospinal tract - from vestibular nuclei of upper medulla/lower pons. Sends ipsilateral neurones down to the anti-gravity muscles to manage posture and balance. 3) Rubrospinal tract - comes from the red nucleus which receives commands from the cerebellum and relays them to the muscles via the spinal cord. Involved in velocity/muscles. Lesion = slowness. 4) Reticulospinal tract - comes from reticular formation in the midbrain/pons - autonomic functions eg. breathing, general arousal. Project bilaterally down spinal cord. 5) Tectospinal tract - minor extra-pyramidal tract. Comes from superior colliculus and is involved in coordination of voluntary eye and head movement in response to visual and auditory stimuli.

Be able to describe the use of memantine and its rationale.

Memantine is an NMDA (N-methyl-D-aspartate) receptor inhibitor. It is thought that glutamate excitotoxicity is an etiological factor for Alzheimer's disease. The glutamate excitotoxicity causes excess Ca2+ to enter neurones via NMDA receptors. The influx harms mitochondria/produces reactive oxygen species --> neuronal damage. Memantine blocks Ca2+ entry though NMDA glutamate channels to prevent this damage taking place. To be used in moderate to severe Alzheimer's Disease.

Describe the acute and chronic deficits arising from lesioning of individual motor tracts and the acute and chronic deficits arising from lesioning the motor cortex. Be able to compare with lesions of the lower motor neurone.

Mentioned in previous learning objectives. At level of motor cortex - discrete, acute lesions cause initial paralysis but can see recovery due to plasticity of brain. Some weakness, clumsiness and fatigue may still be present. A large lesion will cause substantial damage and cannot be recovered from in the same way - slower and incomplete recovery: will get clasp-knife reflex and hemiplegic dystonia. Also amplified weakness, clumsiness and fatigue. At the level of spinal cord - you can get "spinal shock". Acute effects include paralysis or paresis, reduced tendon reflexes in all muscles below level of injury. If there is total transection of spinal cord then these effects will be permanent. Chronic effects include Babinski sign, clonus, slow reappearance of weak monosynaptic reflexes, crossed extensor reflexes. In severe injury, you get hyperactive reflexes - no longer modulated by the brain. Primary motor cortex - paralysis/paresis Premotor cortex - apraxia Frontal eye fields - oculomotor apraxia Dorsolateral prefrontal cortex - apathy and problems with short-term verbal/spatial information (depends if L/R side affected). Orbitofrontal cortex - impulsiveness, puerility, sexual disinhibition etc. Damage to corticospinal tract - above level of medullary pyramids - affects the contralateral side (before decussation). Below, is ipsilateral.

Describe the major neuronal pathways involved in the effects of drugs with dependence potential.

Mesolimbic system: Ventral tegmental area dopaminergic neurones projecting to the nucleus accumbens. This is the brain reward circuitry. Changes that take place in the brain with addiction: Mesolimbic pathway activity after use of the drug increases. Metabolic profile of the brain is hugely decreased (seen on PET scans using fluoro-2-deoxyglucose). There is a negative feedback via the brain reward circuity --> excessive dopamine release --> up regulation of dysnorphin --> dysnorphin peptides bind to kappa opioid receptors and cause reduced release of dopamine --> dysphoria and low mood in addicted patients. This effect spreads throughout the brain affecting VTA, NA, amygdala, insult, prefrontal cortex, hippocampus. The brain exhibits neuroplasticity "rewiring" of synapses which leads to gradual involvement of more and more structures. There becomes a reduced number of D2 dopamine receptors available in the brain. The prefrontal cortex of the brain is involved in many executive functions such as decision making, self-control, personality expression and salience attribution. As this region of the brain degenerates, patient shoes iRISA - impaired response inhibition and salience attribution. Gradual accumulation of gene expression changes eg. alcoholism up regulates NMDA receptor expression. Three stages in drug addiction 1) Intoxication/binging 2) Withdrawal and negative effect 3) Preoccupation or anticipation

Describe how strokes affect multiple cortical regions of the brain.

Middle cerebral artery - frontal lobe, basal ganglia (via lenticulostriate arteries). Damage to the frontal lobes causes contralateral paresis/paralysis. Damage to basal ganglia much more disabling - motor deficit is much greater than if it was just the motor cortex affected alone. Anterior cerebral artery - contralateral paralysis/paresis of lower limbs - affects inner part of homunculus. Motor thalamus damage - severe paralysis/disability as the motor thalamus has inputs to the basal ganglia and cerebellum.

Explain the difference between conditions closely related to the vegetative state such as minimally conscious state and akinetic mutism.

Minimally conscious state is a "shade more aware" than a persistent vegetative state. The patient can be aware of themselves or the environment and smile or vocalise slightly but are still not fully aware/do not form purposeful tasks. Eg. people recovering from TBI. Akinetic mutism - a state of not moving (akinetic) and not speaking (mutism) though the patient is awake and aware. They can move their eyes in response to sound and may whisper monosyllables. Some patients can move in a slow manner if instructed to. They describe it as a resistance meeting them when they want to move or speak. It is caused by a frontal lobe/mesencephalic (midbrain) injury.

Know the pathophysiological changes in depression and the genetic risk factors.

Monoamine deficiency theory: Lowered levels of serotonin, noradrenaline, BDNF (brain derived neurotrophic factor), raised cortisol, pro-inflammatory cytokines and HPA axis dysregulation cause mood changes in depression. Also issues with MAOA iso forms. Anatomical areas affected: mesolimbic system, amygdala, hippocampus, prefrontal cortex, striatal regions, subgenual area (area rich in serotonin transporters). Decreased cortical thickness in depression, thought to be due to raised cortisol and IL-6 inflammatory cytokine. Particular loss is seen in hippocampus. Increased activity in the default mode network. Decreased connectivity between the amygdala/prefrontal cortex. *Increased activity in the amygdala, hippocampus and anterior cingulate cortex which deal with negative stimuli/thoughts --> rumination *Decreased activity from the ventrolateral and dorsolateral prefrontal cortex - decreased cognitive control of these processes.

List the clinical signs of vestibular system dysfunction

Motion sickness - kinetosis Meniere's Disease - tinnitus, vertigo, hearing loss BPPV - benign paroxysmal positional vertigo Signs/symptoms: Nystagmus, dizziness, vertigo, nausea, disequilibrium.

Describe the neuroimmunological aspects of multiple sclerosis.

Multiple sclerosis is an inflammatory demyelinating and degenerative condition.

Describe some common pathologies of sleep and consequences of insomnia

Narcolepsy - rare, long-term brain disorder which causes the individual to fall asleep at inappropriate times. Signs/symptoms: cannot fall asleep and stay asleep, fall asleep when laughing, sleep paralysis, cataplexy, sleep attacks. Pathophysiology: T-lymphocyte mediated autoimmune attack on orexigenic neurones in the brain. Orexins (hypocretin) activate ACh, NA, 5-HT and DA release from the brainstem nuclei to induce arousal and appetite. The orexigenic neurones project to the cerebral cortex, especially the association cortex, raphe nucleus and locus coeruleus. *orexins are too large to be NTMs. Sleep disturbances in Alzheimer's Disease: Damaged nucleus basalis which projects cholinergic signals to the cerebral cortex. Insomnia - memory issues, daytime sleepiness, linked to obesity (leptin released in sleep), depression. Treatments: lifestyle changes, CBT, antihistamines eg. Nytol, benzodiazepines eg. diazepam, temazepam (GABAergic systems more active causing sedation/relaxation), zopiclone and related drugs (Z-drugs). Sleep apnoea - when motor programmes to breathe stop during sleep. Muscles of the throat relax too much or the diaphragm get paralysed due to abnormally powerful inhibition of descending corticospinal tract - patient stops breathing for about a minute. Carbon dioxide builds up and is detected by chemoreceptors which wakes the patient up. Sleepwalking and nocturnal movement - occurs when the descending corticospinal tract is not properly inhibited in sleep.

Understand the role of genetic and environmental factors in headache (thresholds and triggering/precipitating factors)

Non-avoidable risk factors: Genetic threshold for migraine - the number of triggers will vary from person to person. Excitability/sensitivity to stimuli - light, sound, smells, touch. Poor eyesight/declining eyesight Avoidable risk factors: Mental stress, loud noise, insufficient sleep, excess noise, hunger, caffeine, alcohol, reading, shopping, excessive cold, travelling, exercise.

Give a classification of unconscious states.

Normal - awake and aware Locked-In Syndrome - awake and awake but unable to move muscles. Vegetative state - no awareness, but aroused - eyes open but do not look at anything Minimal consciousness state - aroused but only partially aware, may smile sometimes/show discomfort but not all the time. They may watch people move across the room but can't do purposeful tasks like drink water Coma - neither awake nor aroused

Describe opioid and non-opioid analgesic drugs, including specific examples.

Opioid drugs are naturally occurring in the body - for example, enkephalins, dynorphins, beta-endorphin. Opioid receptors: mu, delta, kappa, novice-tin and orphanin. Three endogensous opioid systems to be aware of : 1) proopiomelanocortin-derived 2) proenkephalin-derived 3) pro-dynorphin-derived Morphine: activates potassium conductance, decreases calcium conductance and can cause internalisation of opiate receptors. Overall --> decreased excitability/release of ntms. Morphine is made into very powerful M6G metabolite in the body and must be titrated carefully. Codeine: short acting with 3h half life Methdone: long acting with 24h half life Heroin: high solubility Naloxone: opiate agonist used in palliative care- very short half life Fentanyl: very potent agonist. Tramadol: moderate to severe pain, bind mu receptor and inhibits re-uptake of serotonin and noradrenaline. Opioid switch: swapping patient's opiate medication to another type because they have become tolerant to original prescribed PCA - patient controlled analgesia Non-opioid drugs: Paracetamol: COX-2 inhibitor/modulation of cannabinoid system. Not anti-inflammatory Ibuprofen, diclofenac, ketoprofen, aspirin: COX-1/COX-2 inhibitors Rofecoxib: selective COX-2 inhibitor Amitriptyline: tricyclic antidepressant - inhibits reuptake of amines and blocks sodium/calcium channels. Triptans: sumatriptan --> 5-HT1B receptor for cerebral vessels or 5-HT1D for trigeminal nucleus. Anticonvulsants: carbamazepine, sodium valproate - sodium channel blockers. Stabilises Na+ --> decrease excitability. Pregabalin, gabapentin, verapamil - calcium channel ligands. Pregabalin works on the a2d subunit of calcium channels. Duloxetine - SSRI antidepressant Capsaicin cream - activates TRPV1 receptors *First line treatments for neuropathic pain are antidepressants (duloxetine) followed by tramadol and botox.

Review terms associated with pain systems

Pain - an unpleasant sensory and emotional experience associated with actual or potential tissue damage. Pain is a cerebral construct/perception. Nociceptor - a sensory neurone transducing potentially harmful stimuli. Allodynia - central pain sensitisation normally following often normal non-harmful stimuli Hyperalgesia - an increased sensitivity to pain, possibly caused by damage to nociceptors/peripheral nerves.

How is pain assessed?

Pain scales (1-10, set of facial expressions). SOCRATES approach: site, onset, character, radiation, alleviating factors, time course, exacerbating factors, severity. Chronic pain: McGill Questionnaire - choose words to accurately represent the pain: divides into sensory, cognitive and affective. Pain behaviours: 1) Negative affect - crying 2) Avoidance of certain activities 3) Impaired ambulation/movement 4) Expression of pain - shouting, grimacing

Be able to describe the principal connections between the foregoing limbic structures and the hypothalamus, anterior and torso-medial thalamic nuclei, prefrontal cortex and midbrain reticular formation.

Papez's Circuit (recollective memory): Hippocampus --> mamillary bodies of the hypothalamus via fornix --> anterior thalamus --> anterior cingulate cortex --> parahippocampal gyrus --> back to hippocampus and goes round again. A different set of connections is responsible for fight/flight - from the amygdala --> reticulospinal tract --> sympathetic drive to fight/run: Amygdala --> hypothalamus --> reticular formation --> reticulospinal tract. Role of hypothalamus in limbic system - connects brain to internal systems of the body, connects reticular formation to signal wake/sleep, modulate pain reflexes/analgesia, trigger symp/parasymp activities. Can release hormones from pituitary.

Apply knowledge of basal ganglia to pathologies.

Parkinson's Disease - loss of dopaminergic neurones in the substantia nigra pars compacta of the midbrain. Inability to start/stop motor programmes. Four cardinal symptoms: 1) Resting tremor 2) Bradykinesia 3) Rigidity 4) Loss of postural reflexes Treatments: L-dopa with dopa decarboxylase inhibitors, MAO-B inhibitors, COMT inhibitors, dopamine receptor agonists, amantadine, apomorphine, deep brain stimulation. Huntington's Disease - caused by loss of GABAergic neurones in the striatum (caudate and putamen). The mode of inheritance is autosomal dominant. Huntington's produces opposing signs and symptoms to Parkinson's: 1) Hyperkinesia 2) Ballismus - involuntary jerky movements 3) Athetosis - involuntary smooth movements 4) Tourette's 5) Dementia Treatments for Huntington's Disease include dopamine antagonists (neuroleptics), anti-psychotic drugs and SSRIs/benzodiazepines for depression.

Can describe the pathophysiology and clinical symptoms of movement disorders.

Parkinson's Disease: Loss of dopaminergic neurones in the substantia nigra pars compacta of the midbrain. Lewy bodies form inside the neurones. These are aggregate of the protein alpha-synuclein. Premotor symptoms appear 12-15 years before onset of motor symptoms: sleep disturbances, olfactory dysfunction, dementia, depression, autonomic dysfunction such as dizziness and psychotic symptoms. Motor symptoms: bradykinesia, resting tremor, rigidity, loss of postural reflexes, shuffling gait, micrographia. Causes/risk factors: thought to be a combination of genetic and environmental factors - mutated genes for alpha synuclein PARK1/PARK4. MPTP (methyl-phenyl-tetrahydropyridine) is a neurotoxin metabolised by MAO-B enzyme in the body to create toxic free radicals, these kill dopaminergic terminals. Huntington's Disease: Autosomal dominant inheritance - mutations on chromosome 4, for protein huntingtin - excess repeats for glutamine (>36 CAG repeats) Loss of GABAergic neurones in the striatum, caused by deposits of neurotoxic huntingtin protein. Whole brain atrophies. Signs and symptoms: Hyperkinesia, ballismus, athetosis, Tourette's, dementia.

Explain the concepts of peripheral and central sensitisation and their importance to medicine.

Peripheral sensitisation - for protection and to enable healing. Flare peptidergic C fibres release substance P and CGRP peptides which lower the threshold for nociceptor activation - increased pain response to stimuli. On a molecular level this is due to: 1) sodium channels opening more easily 2) closure of potassium channels 3) TRPV1 channels becoming more sensitive to heat 4) nociceptors becoming tonically active Central sensitisation - prolonged nociceptor input. Nociceptor afferents release glutamate into the dorsal horn. There is an activity dependent increase in the excitability of the dorsal horn projection/relay neurones which outlast the inducing stimulus. There is modified responsiveness of enhanced response to low level inputs including the A-beta fibres which aren't normally pain transmitting. Primary hyperalgesia at the site of flare - increased mechanical and thermal pain. This is due to peripheral sensitisation. Secondary hyperalgesia - sensitive to pain outside of flare region. Mechanical allodynia is also likely to be observed - this is due to central sensitisation.

Symptoms of the mind: depression

Persistent low mood, loss of pleasure in activities previously found pleasurable, poor attention/focus, fatigue, sleep disturbances, insomnia, weight gain, loss or appetite changes, diurnal mood changes, suicidal thoughts or ideation, self harm, loss of libido, feelings of guilt, drug abuse.

Know the signs and symptoms of major depressive disorder

Persistent low mood, loss of pleasure in activities previously pleasurable, suicidal ideation/thoughts of self harm, appetite and weight changes, sleep disturbances or insomnia, lack of motivation, difficulty in concentrating, social withdrawal, adhedonia

Define the terminology used for vegetative states.

Persistent vegetative state > 1 month Permanent vegetative state > 12 months for traumatic or > 6 months for anoxic

Describe the role of prostaglandins in pain.

Phospholipase A2 releases arachidonic acid --> COX-1 and COX-2 enzymes synthesise prostaglandins --> sensitise flare peptidergic C fibres by opening more sodium channels. Drug targets: NSAIDS target the COX enzymes, steroids target phospholipase A2.

Describe the effects of opioids on the body.

Positive: analgesia, sedation Negative: respiratory depression, dysphoria, hypotension, nausea, vomiting, dependence, reduced GI motility and constipation. Recreational: analgesia, sedation, euphoria.

What is the function of prefrontal cortex and temporal lobe?

Prefrontal cortex - involved in executive functions like planning complex cognitive behavior, personality expression, decision making, and moderating social behaviour. Temporal lobe - processing sensory input into derived meanings for the appropriate retention of visual memory, language comprehension, and emotion association.

Understand the difference between primary and secondary headaches.

Primary - diagnosis can be made using the history in the absence of clinical signs. Secondary - the diagnosis can be made using the history in the presence of clinical signs eg. a sinus headache shows green discharge from the nose.

What is the mesocortical system?

Projections from ventral tegmental area to the prefrontal cortex - involved in cognitive control, motivation and emotional response.

Describe how pain is modulated.

Raphe nucleus can send inhibitory signals to the dorsal horn of the spinal cord with neurotransmitters like 5-HT, enkephalins and GABA. The raphe nucleus can be stimulated to do this through periaqueductal grey and the reticular formation (gigantocellular nucleus). Factors that influence pain: genetics, opioidergic and dopamingergic systems, cultural approach to pain, mood (depression), context, condition, peripheral/central sensitisation.

Define in detail pain pathways and their functional roles.

Receptor proteins on nociceptors detect an initial insult: TRPV1, 2, 3 and TREK-1 are for thermal. Chemical are TRPV1, ASIC3. Mechanical are MDEG, TREK-1, DRASIC. Cold is TRPM8. 1) Anterior spinothalamic tract "neo". Innervated mainly by Ad fibres. Interneurones and projection neurones in lamina V and I. Decussation, nerves go up to the more lateral structures of the thalamus: the VPL and VPM (ventral posterolateral and ventral posteromedial nuclei) and VPI and CL nuclei of the reticular and limbic areas. Innervate the primary somatosensory cortex where there is discrimination, localisation and acknowledged intensity of pain. The anterior spinothalamic tract subserves first. 2) Lateral spinothalamic tract. Mainly innervated by C fibres. Synapse with projection neurones in lamina I. Decussation, axons travel in lateral spinothalamic tract. Innervate the more medial/posterior structures of the thalamus: MDvc (mediodorsal ventrocaudal nucleus) and posterior thalamus (POm) (posterior medial sub nucleus) and VMpo (ventral medial posterior nucleus). Innervates the ACC (anterior cingulate cortex) via the MDvc to create emotional/motivational aspect of pain. Innervates the anterior or rostral insulin via the posterior thalamus to create emotion and quality of pain. Collateral projections: spinal cord for reflexes, reticular activating system for arousal and alerting cortex. Periaqueductal grey for modulation of pain (release of 5-HT inhibitory and enkephalins). Amygdala to label the experience with emotional memory via the parabrachial nucleus of the pons.

Review the anatomical features of the cranium relevant to head injury.

Skull - rigid box to protect the brain Meninges - dura, arachnoid and pia Brain - consistency of soft cheese

Describe the stages of sleep and the significance of REM sleep

Sleep cycles consist of slow-wave sleep and REM sleep. One cycle lasts 90 minutes. Slow-wave sleep is time for growth, rejuvenation and rest of musculoskeletal, immune and nervous systems. Growth hormone is released, leptin is released and cortisol levels drop. Autonomic activity eg. heart rate, blood pressure decrease. Wound healing happens and long bones grow (in children). Four stages of slow-wave sleep with increasing EEG amplitudes - it gets harder and harder to wake someone from sleep as you go from stages 1-4. REM sleep is time for memory consolidation (transfer of memories from short-term to long-term), removing junk memories and defragmenting memories. Acetylcholine is responsible for REM sleep as it is the only neurotransmitter released (serotonin and noradrenaline are inactive in the sleep state). The activity in the cholinergic neurones of the pedunculopontine nucleus/nucleus basalis stimulate the cerebral cortex/thalamus to near-waking levels.

Describe the pathophysiologic mechanisms of spasticity, clonus, hyperreflexia and decorticates and decerebrate postures.

Spasticity - pathologically increased muscle tone. Characteristic of upper motor neurone lesions. Spastic muscles often have increased tendon reflexes. Clonus - series of fasciculations in response to stretching of a particular muscle. Represents upper motor neurone lesion. Ankle clonus - 5 or more beats is pathological (cerebral palsy, stroke, spinal cord damage). Hyperreflexia - pathologically brisk tendon reflex Decorticate posturing - a sign of damage in the corticospinal tract in the midbrain. Arms adducted and flexed over the body, feet plantarflexed and internally rotated. Decerebrate posturing - a sign of severe damage to the brain at the level of the upper brainstem - affecting the corticospinal and rubrospinal tracts. Because these tracts aren't working, there is excessive action of the extra-pyramidal system (by disinhibition), especially the vestibulospinal tract. The patient will have their arms adducted and stiffly extended with wrist pronation, finger flexion and plantarflexion of the feet.

Know some theories of pain.

Specificity theory - the idea that single pain nerves automatically send impulses to a "pain centre" in the brain, through the spinal cord. Somewhat true, but does not take into account emotional modulation of pain or the idea of relay neurones. Gate-control theory - argues that there is a "gate" in the spinal cord which can open to allow pain impulses into the brain or not. Activating: focus on pain, anxiety, depression, boredom. Inhibitory: analgesics, counter-stimulation, distractions, pleasure and concentration. Suggests pain is a perception not sensation.

Explain what is meant by the analgesic ladder and how it works.

Start from bottom and work your way up: 3) high efficacy opioid, non-opioid and adjuvant drugs 2) moderate efficacy opioid, non-opioid and adjuvant drugs 1) non-opioids and adjuvant drugs Not used for joint pain

Define ethical issues such as end of life decision.

Subjective and controversial. An appeal to end a person's life must always go through court and it must be shown that the patient is in a permanent vegetative state. The way it is done is to end hydration and nutrition - the starvation period takes about a month. There can be controversy between hospital and family, eg. one patient was deemed to be minimally conscious state but the wife didn't want him to continue his life like that.

List the potential targets for deep brain stimulation in Parkinson's disease.

Subthalamic nucleus Globus pallidus interna Motor thalamus (zona incerta)

Be able to describe the symptoms, progression and prognosis of multiple sclerosis.

Symptoms: optic neuritis, Lhermitte's symptom (electric shock sensation), Uhthoff phenomenon (blurring vision when hot), dysarthria and dysphagia, useless hand, fatigue, cognitive impairment, mood changes. Progression: very varied. "Relapsing-remitting" makes up 85% of cases but you can also get primary progressive where is just continual increase, secondary progressive which is when there are no longer remitting periods, just disease or atypical disease progressions. There will be many subclinical events before the CIS "clinically isolated syndrome" - the first presentation of MS. From then on you get relapsing-remitting pattern normally. Prognosis: MS is not normally fatal - people can have a normal lifespan, but their quality of life can be greatly affected by the symptoms of MS.

Describe the various mechanisms that may underlie epileptic seizures.

Synchronous wave of depolarisation spreading over the brain "Paroxysmal depolarising shift" (don't confuse it with cortical spreading depression!). Normally there are small patterns of discharge but in epilepsy there are some occasional large waves that affect many neurones. Cellular mechanism: 1) Abnormal neuronal excitability (ion channel changes, SCN1B, KCNQ2) 2) Decrease inhibition (loss of GABAergic inhibitory neurones) Neurones within the epileptic focus have burst-firing potentials. There is paroxysmal depolarising shift in which NMDA receptors allow Ca2+ into their cells. Glial cells are involved in transport/clearance of glutamate via EAAT1/EAAT2 transporters. Glial cell abnormalities are suspected in epilepsy.

Be able to provide an overview of the types of intracranial injury which occur and the basic principles of their management after head injury.

TBIs: Concussion - sudden but short-lived loss of mental function that occurs after a blow or other injury to the head. It is the most common but least serious type of brain injury. Contusion - a region of injured tissue/skin in which blood capillaries have been ruptured; bruise Coup-contrecoup injury - when the brain hits the skull on the opposite side of the injury so you get injuries on both sides. Diffuse axonal injury - a brain injury in which damage in the form of extensive lesions in white matter tracts occurs over a widespread area. Penetrating head injury - injury in which dura mater is breached. High-velocity projectiles or low-velocity eg. knives. Management: CT scan to assess, craniotomy, neurointensive care, rehabilitation and physiotherapy. *initially need to decrease intracranial pressure to allow reperfusion - can remove part of the skull.

Review treatments for headaches.

Tension-type headaches are given over-the-counter medication like ibuprofen, paracetamol and aspirin. Cluster headache - treat with high flow oxygen 100% for 12-15l per min, sumatriptan/zolmitriptan (nasal sprays/injections). Should relieve pain within 15-30 mins. How does sumatriptan work? Agonist at 5-HT 1B or 1D receptors on cerebral arteries/veins - reduce vascular inflammation by causing vasodilation. Coca-cola? Water and sugar.

Describe clinical tests and the diagnostic criteria for multiple sclerosis.

Tests: Scanning for lesions of T2/flair scans. The lesions should be disseminated in time and space - e.g. corpus callous and periventricular lesions are diagnostic of MS. Brain atrophy will be present - more marked than in normal ageing. Marco-enlargement of the brain ventricles. CSF extracted with an atraumatic needle from subarachnoid space shows oligoclonal bands after gel electrophoresis. The immunoglobulin ratio in CSF to serum is also raised. Can look for neurofilaments in CSF. Diagnostic criteria: 1) Neurologic defects lasting greater than 24 hours 2) Multiple lesions disseminated in time 3) Multiple lesions disseminated in space 4) Exclusion of other causes of CNS malfunction e.g. syphilis, infections, B12 deficiency.

Know the classification of mood disorders.

The DSM-IV (diagnostic and statistical manual of mental disorders) enables you to diagnose four mental health conditions: major depressive disorder, bipolar disorder, dysthymic disorder, depressive disorder not otherwise specified.

Revise the location of the basal ganglia in relation to the ventricles. Locate basal ganglia structures.

The caudate lies along the concave aspect of the anterior part of the lateral ventricle. The caudate curves round to end in the temporal lobes. The putamen is attached to the caudate by cell bridges and is a bean shape, more lateral than the caudate. It sits in the centre of the curve of the lateral ventricle. Together the caudate and the putamen are the dorsal striatum. The ventral striatum is made of the nucleus accumbens which is a small oval shape on the anterior side of putamen. The globus pallidus are like wedges of cheese internal to the putamen. The substantia nigra is found in the base of the midbrain. The subthalamic nucleus is found below the thalamus.

Describe the EEG and its significance

The electroencephalograph records microvolts from the surface of the scalp to show the activity of thousands of neurones at once. Recording are made from pairs of electrodes at different positions on the scalp. The more neurones firing at once, the greater the amplitude of the EEG waves. It is best used to record seizures, sleep and identify comas. During waking, neurones are firing asynchronously and the EEG shows small desynchronised amplitudes. In contrast, during sleep, many neurones fire synchronously, producing large waves amplitudes on the EEG. In a comatose patient you will see no waves on EEG. The EEG tells us that consciousness comes from the cerebral cortex. It has been noticed that during some stages of sleep (REM), the EEG looks identical to that of an alert, aroused person - therefore there are some questions about sleep states in terms of consciousness.

What is neuroadaption?

The process why which the brain alters its chemistry in response to presence of certain chemicals. In addiction, the more drug is taken, the more the brain chemistry changes to compensate for drug presence. Then tolerance develops --> leads to addiction and dependence.

Describe the vestibulo-ocular reflex and explain how nystagmus can occur

The semicircular canals get activated by head rotation and send their impulses via the vestibular nerve (cranial nerve VIII) to the vestibular nuclei in the brainstem. From these nuclei, fibers cross to the contralateral cranial nerve VI nucleus (abducens nucleus) and contralateral oculomotor nucleus. One pathway projects directly to the lateral rectus of eye via the abducens nerve. The other activates the medial rectus muscle of the eye through the oculomotor nerve. When you stop spinning, endolymph keeps moving causing the vestibulo-ocular reflex. This is normal "optikokinetic" nystagmus. Spontaneous nystagmus occurs when there is damage to the vestibular apparatus, brainstem or cerebellum.

Understand the physiological principles relating to the intracranial pressure.

The skull cannot change size so bleeds, swelling, inflammation in the brain can cause the intracranial pressure to rise. The Munro-Kelly doctrine explains the relationship between intracranial content and intracranial pressure. If you add mass/oedema: You can squeeze out CSF into the spine/venous blood through venous ostea of skull. However, once these mechanisms are exhausted the ICP rises. What happens when the intracranial pressure rises: The cerebral perfusion pressure (CPP) drops (difference between MAP-ICP). As the ICP goes up, the CPP goes down and this is the effective perfusion pressure in the brain. The brain will start to infarct when ICP rises too much. The brain will be squashed and squeezed into other places - herniation. Axonal transport disrupted --> stops normal brain function. Herniation types: 1) subfalcine - cerebral cortex goes under the falx cerebri 2) uncal - the cerebral cortex goes under the tentorium cerebelli. Damage to the oculomotor nerve: dilated and fixed pupil. 3) tonsillar - the cerebellum goes out of the foramen magnum

List the functional roles of the vestibular system

The vestibular system is involved in balance, posture and spatial orientation. There are three integrated components of the vestibular system: visual input from the eyes, proprioceptive and cutaneous receptors in the muscles and vestibular receptors in the inner ear. Inside the inner ear you can find the vestibular system and the auditory system.

Describe the signs and symptoms of schizophrenia

Three categories of symptoms: 1) Positive - hallucinations, delusions, disorganised behaviour, agitation 2) Negative - social withdrawal, apathy, emotional blunting, poverty of speech, neglect of personal care 3) Cognitive - poor memory, lack of concentration, executive dysfunction DSM IV (diagnostic and statistical manual of mental disorders) can be used to diagnose schizophrenia by recording all different types of symptoms onto a graph. If cognitive dysfunction is bad the patient may be asked to do Wisconsin scoring test.

Describe how consciousness is measured using the Glasgow coma scale

Three categories: EYES (1-4), VERBAL (1-5), MOTOR (1-6). 1 = normal, the last value for each = none. Eyes: E1 spontaneous, E2 to sound, E3 to pressure, E4 none. Verbal: V1 orientated, V2 confused, V3 words, V4 sounds, V5 none. Motor: M1 obeys commands, M2 localising, M3 normal flexion, M4 abnormal flexion, M5 extension, M6 none. *Patients in persistent vegetative state will have E1 - they open and close their eyes in circadian rhythms but have no awareness of themselves or their surroundings - V5/M6.

Explain the prognosis and outcome of vegetative state.

Three factors affect prognosis: 1) Time spent in vegetative state: best factor for predicting prognosis. After 6 months, chance of waking is only 3%. 2) Age of patient: below 20 years, 21% gain independence again. Above 40 years, 0% gain independence. 3) Type of injury: hypoxic have poor prognosis. Treatments can be given to try and wake someone from a veg state: Zolpidem targets the GPi and inhibits it so that the patient can wake up. Amantidine is another drug choice (NMDA receptor antagonist, increases release of dopamine, prevents dopamine reuptake). Deep brain stimulation of the central thalamus is a non-pharmacological treatment possiblity Future: electrode stimulation of motor cortex, modulation of brain stream.

Understand the distinguishing of symptoms of the primary headache.

Three types: 1) Tension-type - 80% of all primary headaches. Feels like a tight band around the forehead, can radiate to neck and shoulders. Lasts from 30mins to several days. Does not change with movement. 2) Migraine - 15% of all primary headaches. Aura is virtually always visual - a bright blob growing over the vision, it takes 20-30 minutes to develop each symptom. Caused by cortical spreading depression - chemical changes which precipitate a migraine happen 24 hours before the migraine itself. 3) Cluster headache - most common in males. 0.2-0.3% of all primary headaches. Excruciating pain above or around one eye. Recur in 30min-2h bouts for several weeks, the patient goes into remission for months or even years after. Poorly diagnosed and not well understood.

Explain transduction, transmission, perception and modulation in the nociceptive system.

Transduction - when pain is changed from mechanical/temperature stimulus to electrical activity. Transmission - the relay of the electrical impulse to the central nervous system (spinal cord and brain) Perception - discrimination, affect and motivation of pain Modulation - the previous three steps are modulated positively or negatively affecting the perception of pain. Each of TTPM is a possibly drug target.

Know the types of antidepressant drugs, their mechanisms of action and adverse side effects.

Tricyclic antidepressants: amitriptyline, imipramine, clomipramine, protriptyline. Mechanism: Prevent uptake of amines 5-HT and NA, can also work at histamine receptors (sleep enhancing) and adrenergic receptors. SFX: Cardiotoxicity in overdose. Other: dry mouth, constipation, weight gain, sedation, urinary retention, loss of libido, postural hypotension. MAO-A/B inhibitors - Iproniazid, phenelzineu, tranylcypromine - irreversibly inhibit MAO-A/MAO-B enzymes to prevent breakdown of noradrenaline/dopamine in the body. SFX: hepatotoxicity. Interacts with tyramine-containing foods "cheese effect" - cheese, red wine, pickled fish and eggs. SSRIs - selective serotonin reuptake inhibitors, fluoxetine, citalopram, paroxetine - increased selectivity for serotonin reuptake. SAFE IN OD. SFX: sexual dysfunction, nausea, headaches, GI issues, anxiety, insomnia, headaches Reversible MAO inhibitors - Moclobemide, increased selectivity for MAO-A. Safer than irreversible. SFX: confusion, agitation, nausea. SNRI - serotonin noradrenaline reuptake inhibitor - Venlafaxine, fluoxetine. NaRI - noradrenaline reuptake inhibitor - Reboxetine NaSSA - Mirtazapine - noradrenaline and specific serotonergic antidepressant SARI - serotonin antagonist and reuptake inhibitor - trazodone *SNRI/NaRI/NaSSA/SARI all have fewer effects than MAOi, SSRI, TCAs. Melatonin agonist - agomelatine - agonist at MT1 and Mt 2 receptors, antagonist at 5-HT receptors. Improves sleep quality, anxiolytic effects, no sexual dysfunction, no discontinuation syndrome.

Know the treatment of trigeminal neuralgia.

Trigeminal neuralgia is the most common facial pain syndrome. Sudden, paroxysmal sharp pain which feels like electric shocks or stabbing. Can be unilateral. Lasts from a few seconds to a few minutes. Treatment is ABC: Anticonvulsants: carbamazepine, sodium valproate, phenytoin Benzodiazepines: diazepam, clonazepam CNS depressants (Baclofen, GABAr agonist)

Describe the pharmacology of schizophrenia

Typical vs. atypical antipsychotics. Typical antipsychotics are first generation, all work at 5-HT2 receptors, but chlorpromazine and thioridazine have a notable higher affinity for a-1 noradrenergic receptors. Flupenthixol works at D1 dopamine receptors also. Haloperidol, chlorpromazine (first antipsychotic for anxiety, bipolar disorder, behavioural issues), fluphenazine, flupenthixol, thioridazine (withdrawn for negative cardiac effects). Flupenthixol is a long-acting injection for poorly compliant patients, also a low-dose antidepressant. SFX: weight gain, dry mouth, muscle cramping, muscle stiffness, hyperprolactinaemia, QT interval prolongation (cardiac toxicity), sudden death. Neuroleptic malignant syndrome - high fever, muscle rigidity, tremors, autonomic instability, altered mental status (delirium). Atypical antipsychotics are second generation and should be prescribed first in accordance with NICE guidelines as they have fewer side effects, better efficacy, are less addictive and are less likely to cause extrapyramidal dysfunction and motor side effects. Olanzapine, clozapine, risperidone, quetiapine, apiprazole, paliperidone. Have antagonistic effects at 5-HT2 receptors. May mildly improve cognition over time. Clozapine blocks D4 receptors with high affinity and is associated with agranulocytosis (severely lowered WBC count). SFX: weight gain, dry mouth, muscle cramping, muscle stiffness, insulin resistance, hyperglycaemia, diabetes, dyslipidaemia, cardiovascular disease. A >80% blockade of D2 receptors can cause extrapyramidal dysfunction - acute dystonia, parkinsonism, tardive dyskinesia - this is because the effect is like Parkinson's Disease loss of dopaminergic neurones. Non-pharmacological: CBT, family therapy, group therapy. Compliance is problematic - social activities benefit schizophrenic patients greatly.

Briefly review future directions in the management of Parkinson's Disease and Huntington's Disease.

Use of embryonic mesencephalic grafting - proven to be fully functional in Parkinson's patients when imaged with raclopride/stimulated by metamphetamine. But striatal foetal grafts degenerate in Huntington's patients due to huntingtin build up - same pathology as in the original brain tissue.

Have an overview of various treatments for addiction

Varenicline - a4b2 selective partial agonist at nicotinic receptors. Nicotine replacement therapies. Nalmefene can be used to taper off alcohol consumption/opiate consumption (opiate agonist) Disulfiram (antabuse) is an aldehyde dehydrogenase inhibitor - allows accumulation of toxic product of alcohol acetaldehyde to accumulate which makes the patient vomit - is an aversion therapy. *alcohol is one of the most dangerous drugs to stop taking suddenly - delirium tremens is an extreme consequence - a psychotic condition typical of withdrawal in chronic alcoholics, involving tremors, hallucinations, anxiety and disorientation. To treat withdrawal symptoms: SSRIs, MAOAs (bupropion), opioid receptor antagonists (naltrexone).

Describe the main gross anatomical features of the cerebellum, including its input and output pathways and its lobular arrangement.

Vermis - line runs down the cerebellum sagittally, but does not separate it into two lobes - cerebellum is one walnut. Lots of small gyri. Folia appearance - oak leaves. Remember cerebellum is kind of at ponto-medullary junction. Anterior and posterior lobes are separated by the primary fissure. Flocculus on the anterior side makes up the flocculo-nodular lobe (connected to/same functions as the vestibular nucleus - balance, posture, head/eye coordination). Flocculus makes up floor of 4th ventricle. Blood supply - PICA (posterior inferior cerebellar artery) and basilar branches (AICA and SCA). Cerebellar peduncles come off the brainstem: 1) superior cerebellar peduncle - output tract 2) middle cerebellar peduncle - input from contralateral cortex 3) inferior cerebellar peduncle - input from spinal cord/spinocerebellar tracts. Spinocerebellar tract is for proprioception of joints and muscles.

Describe the functional significance of cerebellar connections to the vestibular system.

Vestibulocerebellum - works with the medial and lateral vestibulospinal tracts (which come from medial lateral fasciculus and vestibular nuclei respectively). The flocculonodular lobe stores motor programmes for balance/posture and head/eye coordination in tracking.

Describe the central pathways of the vestibular system

Vestibulocerebellum/vestibulospinal tracts.

What is disinhibition?

When two inhibitory neurones placed in tandem overall create the effect of excitation. This "excitation by double inhibition" is disinhibition.


Conjuntos de estudio relacionados

Combo with "Evolve: Psychobiological Disorders" and 8 others

View Set

Leadership & Management - UWorld

View Set

Combo with "TAM 1200 FINAL EXAM" and 11 others

View Set

Chapter 41: Gastrointestinal Dysfunction

View Set

Chapter Exam - Life Underwriting

View Set

Chapter 22, Section 1 : Life in the industrial Age

View Set

CLEP Human Growth and Development

View Set

Cell and Molecular Biology, Chapter 1

View Set

Cog Psych Chapter 10, Cog Psych Chapter 11, Chapter 12, Cog Psych Chapter 13

View Set